Udowodnij nierówność

Podzielność. Reszty z dzielenia. Kongruencje. Systemy pozycyjne. Równania diofantyczne. Liczby pierwsze i względnie pierwsze. NWW i NWD.
Bandysc
Użytkownik
Użytkownik
Posty: 34
Rejestracja: 16 wrz 2012, o 19:35
Płeć: Mężczyzna
Lokalizacja: Łódź
Podziękował: 10 razy

Udowodnij nierówność

Post autor: Bandysc »

Znajdź najmniejszą spośród liczb naturalnych, takich, że dla każdej liczby naturalnej \(\displaystyle{ n \in­ N}\) zachodzi \(\displaystyle{ 6 + \frac{4^n}{2\sqrt{n}} \le {2n \choose n}}\).

"Ręcznie" udało mi się znaleźć, że to równanie jest spełnione dla \(\displaystyle{ n \ge 4}\), niestety nie wiem jak to udowodnić \(\displaystyle{ n}\) w silni, pierwiasteku, potędze... Czy ktoś ma jakiś pomysł? Z góry dziękuję za pomoc
Awatar użytkownika
musialmi
Użytkownik
Użytkownik
Posty: 3466
Rejestracja: 3 sty 2014, o 13:03
Płeć: Mężczyzna
Lokalizacja: PWr ocław
Podziękował: 382 razy
Pomógł: 434 razy

Udowodnij nierówność

Post autor: musialmi »

Zazwyczaj próbuje się indukcyjnie, chociaż pierwiastek chyba nie ma zamiaru tego ułatwiać.
Zahion
Moderator
Moderator
Posty: 2095
Rejestracja: 9 gru 2012, o 19:46
Płeć: Mężczyzna
Lokalizacja: Warszawa, mazowieckie
Podziękował: 139 razy
Pomógł: 504 razy

Udowodnij nierówność

Post autor: Zahion »

Bandysc pisze:"Ręcznie" udało mi się znaleźć, że to równanie jest spełnione dla n ge 4,
Równanie, a na dodatek ręcznie, dużo liczenia
Indukcja, pierwszy krok miejmy za sobą.
Dowodzimy dla \(\displaystyle{ n + 1}\).
Mamy otóz \(\displaystyle{ {2n+2 \choose n+1} = \frac{\left( 2n\right)! }{n!n!} \cdot \frac{\left( 2n+1\right)\left( 2n+2\right) }{\left( n+1\right)^{2} } \ge \left(6+ \frac{4^{n}}{2 \sqrt{n} } \right)\left( \frac{4n+2}{n+1} \right)}\).
Udowodnijmy teraz, że \(\displaystyle{ \frac{4^{n}}{2 \sqrt{n} } \cdot \frac{4n+2}{n+1} \ge \frac{4^{n+1}}{2 \sqrt{n+1} }}\), co daje nam równoważnie
\(\displaystyle{ \sqrt{n+1}\left( 2n+1\right) \ge 2\left( n+1\right) \sqrt{n}}\). czyli po podniesieniu do kwadratu i wymnożeniu \(\displaystyle{ 4n^{3} + 8n^{2} +5n + 1 \ge 4n^{3} + 8n^{2} + 4n}\) co jest oczywiście prawdą.
Otóż
\(\displaystyle{ \left(6+ \frac{4^{n}}{2 \sqrt{n} } \right)\left( \frac{4n+2}{n+1} \right) = \frac{24n+12}{n+1} + \frac{4^{n}}{2 \sqrt{n} } \cdot \frac{4n+2}{n+1} \ge \frac{24n+12}{n+1} + \frac{4^{n+1}}{2 \sqrt{n+1} } \ge 6 + \frac{4^{n+1}}{2 \sqrt{n+1} }}\),

co kończy dowód indukcyjny.

Wystarczy teraz znalezć wartość z pierwszego kroku.
Bandysc
Użytkownik
Użytkownik
Posty: 34
Rejestracja: 16 wrz 2012, o 19:35
Płeć: Mężczyzna
Lokalizacja: Łódź
Podziękował: 10 razy

Udowodnij nierówność

Post autor: Bandysc »

Oczywiście "nierówność" I faktycznie pięknie z indukcji wychodzi. Dziękuję bardzo za pomoc!
ODPOWIEDZ